Show that $lim_{ntoinfty} int_{0}^{1}f(x)g(nx)dx = 0$ $g$ periodic on $mathbb{R}$












0












$begingroup$


Let $g$ be a continuous periodic function on $mathbb{R}$ with $g(x + 1) = g(x)$. Assume that $int_{0}^{1}g(x)dx = 0$.



(a) Let $f$ be continuous on all $mathbb{R}$



Show that $lim_{ntoinfty} int_{0}^{1}f(x)g(nx)dx = 0$



(b) Let $f$ be Lebesgue integrable on all $mathbb{R}$



Show that $lim_{ntoinfty} int_{0}^{1}f(x)g(nx)dx = 0$



My attempt:



(a)



For part (a) I dont know how to proceed.
I have a solution that uses the following:



$$lim_{ntoinfty} int_{0}^{1}f(x)g(nx)dx = int_{0}^{1}f(x)int_{0}^{1}g(x)dx$$ and by hypothesis the results follow. There is another approach ?



(b) (This needs verification)



$g(x)$ is continuous and periodic on $mathbb{R}$, then $exists B > 0$ such that $|g(x)|< B$.



$f$ Lebesgue integrable on $[0,1]$ $rightarrow forall epsilon > 0, exists h$ step function such that $int_{0}^{1}|f - h| < frac{epsilon}{B}, forall x in [0,1]$



then



$$| int_{0}^{1}f(x)g(nx)dx | leq int_{0}^{1}|f(x) - h(x)| |g(nx)|dx + |int_{0}^{1} h(x)g(nx)dx| leq epsilon + |int_{0}^{1} h(x)g(nx)dx|$$



We must show that $lim_{nto infty}int_{0}^{1} h(x)g(nx)dx|$



Since $h$ is a step function,



$$h(x) = sum_{i=1}^{k}lambda_ichi_{O_i}(x), mbox{ where } bigcup_{i=1}^{k}O_i = [0,1]$$



Hence, for $M = max_{i = 1,...,k}|lambda_i|$



$$int_{0}^{1} h(x)g(nx)dx = sum_{i=1}^{k}lambda_iint_{O_i}g(nx)dx$$
$$leq Msum_{i=1}^{k}int_{O_i}g(nx)dx$$
$$= Mint_{0}^{1}g(nx)dx| = frac{M}{n}int_{0}^{n}g(x)dx = Mint_{0}^{n}g(x) = 0$$



Then



$$|int_{0}^{1}f(x)g(nx)dx| < epsilon$$










share|cite|improve this question









$endgroup$








  • 2




    $begingroup$
    For $(b)$, you can approximate $f$ by continuous $h$ and use the result of $(a)$.
    $endgroup$
    – Song
    Jan 7 at 17:26
















0












$begingroup$


Let $g$ be a continuous periodic function on $mathbb{R}$ with $g(x + 1) = g(x)$. Assume that $int_{0}^{1}g(x)dx = 0$.



(a) Let $f$ be continuous on all $mathbb{R}$



Show that $lim_{ntoinfty} int_{0}^{1}f(x)g(nx)dx = 0$



(b) Let $f$ be Lebesgue integrable on all $mathbb{R}$



Show that $lim_{ntoinfty} int_{0}^{1}f(x)g(nx)dx = 0$



My attempt:



(a)



For part (a) I dont know how to proceed.
I have a solution that uses the following:



$$lim_{ntoinfty} int_{0}^{1}f(x)g(nx)dx = int_{0}^{1}f(x)int_{0}^{1}g(x)dx$$ and by hypothesis the results follow. There is another approach ?



(b) (This needs verification)



$g(x)$ is continuous and periodic on $mathbb{R}$, then $exists B > 0$ such that $|g(x)|< B$.



$f$ Lebesgue integrable on $[0,1]$ $rightarrow forall epsilon > 0, exists h$ step function such that $int_{0}^{1}|f - h| < frac{epsilon}{B}, forall x in [0,1]$



then



$$| int_{0}^{1}f(x)g(nx)dx | leq int_{0}^{1}|f(x) - h(x)| |g(nx)|dx + |int_{0}^{1} h(x)g(nx)dx| leq epsilon + |int_{0}^{1} h(x)g(nx)dx|$$



We must show that $lim_{nto infty}int_{0}^{1} h(x)g(nx)dx|$



Since $h$ is a step function,



$$h(x) = sum_{i=1}^{k}lambda_ichi_{O_i}(x), mbox{ where } bigcup_{i=1}^{k}O_i = [0,1]$$



Hence, for $M = max_{i = 1,...,k}|lambda_i|$



$$int_{0}^{1} h(x)g(nx)dx = sum_{i=1}^{k}lambda_iint_{O_i}g(nx)dx$$
$$leq Msum_{i=1}^{k}int_{O_i}g(nx)dx$$
$$= Mint_{0}^{1}g(nx)dx| = frac{M}{n}int_{0}^{n}g(x)dx = Mint_{0}^{n}g(x) = 0$$



Then



$$|int_{0}^{1}f(x)g(nx)dx| < epsilon$$










share|cite|improve this question









$endgroup$








  • 2




    $begingroup$
    For $(b)$, you can approximate $f$ by continuous $h$ and use the result of $(a)$.
    $endgroup$
    – Song
    Jan 7 at 17:26














0












0








0





$begingroup$


Let $g$ be a continuous periodic function on $mathbb{R}$ with $g(x + 1) = g(x)$. Assume that $int_{0}^{1}g(x)dx = 0$.



(a) Let $f$ be continuous on all $mathbb{R}$



Show that $lim_{ntoinfty} int_{0}^{1}f(x)g(nx)dx = 0$



(b) Let $f$ be Lebesgue integrable on all $mathbb{R}$



Show that $lim_{ntoinfty} int_{0}^{1}f(x)g(nx)dx = 0$



My attempt:



(a)



For part (a) I dont know how to proceed.
I have a solution that uses the following:



$$lim_{ntoinfty} int_{0}^{1}f(x)g(nx)dx = int_{0}^{1}f(x)int_{0}^{1}g(x)dx$$ and by hypothesis the results follow. There is another approach ?



(b) (This needs verification)



$g(x)$ is continuous and periodic on $mathbb{R}$, then $exists B > 0$ such that $|g(x)|< B$.



$f$ Lebesgue integrable on $[0,1]$ $rightarrow forall epsilon > 0, exists h$ step function such that $int_{0}^{1}|f - h| < frac{epsilon}{B}, forall x in [0,1]$



then



$$| int_{0}^{1}f(x)g(nx)dx | leq int_{0}^{1}|f(x) - h(x)| |g(nx)|dx + |int_{0}^{1} h(x)g(nx)dx| leq epsilon + |int_{0}^{1} h(x)g(nx)dx|$$



We must show that $lim_{nto infty}int_{0}^{1} h(x)g(nx)dx|$



Since $h$ is a step function,



$$h(x) = sum_{i=1}^{k}lambda_ichi_{O_i}(x), mbox{ where } bigcup_{i=1}^{k}O_i = [0,1]$$



Hence, for $M = max_{i = 1,...,k}|lambda_i|$



$$int_{0}^{1} h(x)g(nx)dx = sum_{i=1}^{k}lambda_iint_{O_i}g(nx)dx$$
$$leq Msum_{i=1}^{k}int_{O_i}g(nx)dx$$
$$= Mint_{0}^{1}g(nx)dx| = frac{M}{n}int_{0}^{n}g(x)dx = Mint_{0}^{n}g(x) = 0$$



Then



$$|int_{0}^{1}f(x)g(nx)dx| < epsilon$$










share|cite|improve this question









$endgroup$




Let $g$ be a continuous periodic function on $mathbb{R}$ with $g(x + 1) = g(x)$. Assume that $int_{0}^{1}g(x)dx = 0$.



(a) Let $f$ be continuous on all $mathbb{R}$



Show that $lim_{ntoinfty} int_{0}^{1}f(x)g(nx)dx = 0$



(b) Let $f$ be Lebesgue integrable on all $mathbb{R}$



Show that $lim_{ntoinfty} int_{0}^{1}f(x)g(nx)dx = 0$



My attempt:



(a)



For part (a) I dont know how to proceed.
I have a solution that uses the following:



$$lim_{ntoinfty} int_{0}^{1}f(x)g(nx)dx = int_{0}^{1}f(x)int_{0}^{1}g(x)dx$$ and by hypothesis the results follow. There is another approach ?



(b) (This needs verification)



$g(x)$ is continuous and periodic on $mathbb{R}$, then $exists B > 0$ such that $|g(x)|< B$.



$f$ Lebesgue integrable on $[0,1]$ $rightarrow forall epsilon > 0, exists h$ step function such that $int_{0}^{1}|f - h| < frac{epsilon}{B}, forall x in [0,1]$



then



$$| int_{0}^{1}f(x)g(nx)dx | leq int_{0}^{1}|f(x) - h(x)| |g(nx)|dx + |int_{0}^{1} h(x)g(nx)dx| leq epsilon + |int_{0}^{1} h(x)g(nx)dx|$$



We must show that $lim_{nto infty}int_{0}^{1} h(x)g(nx)dx|$



Since $h$ is a step function,



$$h(x) = sum_{i=1}^{k}lambda_ichi_{O_i}(x), mbox{ where } bigcup_{i=1}^{k}O_i = [0,1]$$



Hence, for $M = max_{i = 1,...,k}|lambda_i|$



$$int_{0}^{1} h(x)g(nx)dx = sum_{i=1}^{k}lambda_iint_{O_i}g(nx)dx$$
$$leq Msum_{i=1}^{k}int_{O_i}g(nx)dx$$
$$= Mint_{0}^{1}g(nx)dx| = frac{M}{n}int_{0}^{n}g(x)dx = Mint_{0}^{n}g(x) = 0$$



Then



$$|int_{0}^{1}f(x)g(nx)dx| < epsilon$$







integration measure-theory lebesgue-integral periodic-functions






share|cite|improve this question













share|cite|improve this question











share|cite|improve this question




share|cite|improve this question










asked Jan 7 at 17:18









Richard ClareRichard Clare

1,054314




1,054314








  • 2




    $begingroup$
    For $(b)$, you can approximate $f$ by continuous $h$ and use the result of $(a)$.
    $endgroup$
    – Song
    Jan 7 at 17:26














  • 2




    $begingroup$
    For $(b)$, you can approximate $f$ by continuous $h$ and use the result of $(a)$.
    $endgroup$
    – Song
    Jan 7 at 17:26








2




2




$begingroup$
For $(b)$, you can approximate $f$ by continuous $h$ and use the result of $(a)$.
$endgroup$
– Song
Jan 7 at 17:26




$begingroup$
For $(b)$, you can approximate $f$ by continuous $h$ and use the result of $(a)$.
$endgroup$
– Song
Jan 7 at 17:26










2 Answers
2






active

oldest

votes


















1












$begingroup$

For part (a):
Let $I=int_{0}^{1}|g(x)|dx$. Let $epsilon>0$ be given. Note that
$f$ is uniformly continuous on $[0,1]$, so there exists $delta>0$
such that $|f(x)-f(y)|<frac{epsilon}{I+1}$ whenever $x,yin[0,1]$
with $|x-y|<delta$. Choose $Ninmathbb{N}$ such that $frac{1}{N}<delta$.
By considering the substitution $y=nx$, we may rewrite
begin{eqnarray*}
int_{0}^{1}f(x)g(nx)dx & = & frac{1}{n}int_{0}^{n}f(frac{y}{n})g(y)dy\
& = & frac{1}{n}sum_{k=1}^{n}int_{k-1}^{k}f(frac{y}{n})g(y)dy.
end{eqnarray*}

Let $ngeq N$ be arbitrary. Denote $f_{k}=f(frac{k}{n})$, for $k=1,2,ldots n$.
By periodicity of $g$, we have $int_{k-1}^{k}g(y)dy=int_{0}^{1}g(y)dy=0$.
Therefore
begin{eqnarray*}
& & |int_{k-1}^{k}f(frac{y}{n})g(y)dy|\
& = & |int_{k-1}^{k}f(frac{y}{n})-f_{k}g(y)dy|\
& leq & int_{k-1}^{k}|f(frac{y}{n})-f_{k}||g(y)|dy\
& leq & int_{k-1}^{k}frac{epsilon}{I+1}|g(y)|dy\
& = & frac{epsilon}{I+1}int_{0}^{1}|g(y)|dy\
& = & epsiloncdotfrac{I}{I+1}\
& leq & epsilon.
end{eqnarray*}

Now it is clear that
begin{eqnarray*}
& & |int_{0}^{1}f(x)g(nx)dx|\
& leq & frac{1}{n}sum_{k=1}^{n}|int_{k-1}^{k}f(frac{y}{n})g(y)dy|\
& leq & frac{1}{n}sum_{k=1}^{n}epsilon\
& = & epsilon.
end{eqnarray*}






share|cite|improve this answer









$endgroup$













  • $begingroup$
    how you obtain this:begin{eqnarray*} & & |int_{k-1}^{k}f(frac{y}{n})g(y)dy|\ & = & |int_{k-1}^{k}f(frac{y}{n})-f_{k}g(y)dy| *****\ & leq & int_{k-1}^{k}|f(frac{y}{n})-f_{k}||g(y)|dy\ & leq & int_{k-1}^{k}frac{epsilon}{I+1}|g(y)|dy\ & = & frac{epsilon}{I+1}int_{0}^{1}|g(y)|dy\ & = & epsiloncdotfrac{I}{I+1}\ & leq & epsilon. end{eqnarray*}
    $endgroup$
    – Richard Clare
    Jan 7 at 18:05












  • $begingroup$
    $int_{k-1}^k f_k g(y) dy =0$
    $endgroup$
    – Danny Pak-Keung Chan
    Jan 7 at 18:11










  • $begingroup$
    Sure! Thanks!!!!!!!!
    $endgroup$
    – Richard Clare
    Jan 7 at 18:14



















1












$begingroup$

For (b): Since $g$ is continuous, there exists $M>0$ such that $|g(x)|leq M$
for all $xin[0,1]$. By periodicity of $g$, it follows that $|g(x)|leq M$
for all $xinmathbb{R}$. Recall that Lebesgue integrable function
can be approximated by integrable continuous function in $||cdot||_{1}$-norm.
More precisely, given $epsilon>0$, there exists a continuous function
$h:mathbb{R}rightarrowmathbb{R}$ such that $int_{0}^{1}|f-h|<epsilon$.



Let $epsilon>0$ be given. Choose continuous function $h:mathbb{R}rightarrowmathbb{R}$
such that $int_{0}^{1}|f-h|<frac{epsilon}{2M}.$ By part (a), there
exists $N$ such that $|int_{0}^{1}h(x)g(nx)dx|<frac{epsilon}{2}$
whenever $ngeq N$. Now let $ngeq N$ be aribitrary, then
begin{eqnarray*}
& & |int_{0}^{1}f(x)g(nx)dx|\
& leq & |int_{0}^{1}[f(x)-h(x)]g(nx)dx|+|int_{0}^{1}h(x)g(nx)dx|\
& leq & Mint_{0}^{1}|f(x)-h(x)|dx+frac{epsilon}{2}\
& < & epsilon.
end{eqnarray*}






share|cite|improve this answer









$endgroup$













    Your Answer





    StackExchange.ifUsing("editor", function () {
    return StackExchange.using("mathjaxEditing", function () {
    StackExchange.MarkdownEditor.creationCallbacks.add(function (editor, postfix) {
    StackExchange.mathjaxEditing.prepareWmdForMathJax(editor, postfix, [["$", "$"], ["\\(","\\)"]]);
    });
    });
    }, "mathjax-editing");

    StackExchange.ready(function() {
    var channelOptions = {
    tags: "".split(" "),
    id: "69"
    };
    initTagRenderer("".split(" "), "".split(" "), channelOptions);

    StackExchange.using("externalEditor", function() {
    // Have to fire editor after snippets, if snippets enabled
    if (StackExchange.settings.snippets.snippetsEnabled) {
    StackExchange.using("snippets", function() {
    createEditor();
    });
    }
    else {
    createEditor();
    }
    });

    function createEditor() {
    StackExchange.prepareEditor({
    heartbeatType: 'answer',
    autoActivateHeartbeat: false,
    convertImagesToLinks: true,
    noModals: true,
    showLowRepImageUploadWarning: true,
    reputationToPostImages: 10,
    bindNavPrevention: true,
    postfix: "",
    imageUploader: {
    brandingHtml: "Powered by u003ca class="icon-imgur-white" href="https://imgur.com/"u003eu003c/au003e",
    contentPolicyHtml: "User contributions licensed under u003ca href="https://creativecommons.org/licenses/by-sa/3.0/"u003ecc by-sa 3.0 with attribution requiredu003c/au003e u003ca href="https://stackoverflow.com/legal/content-policy"u003e(content policy)u003c/au003e",
    allowUrls: true
    },
    noCode: true, onDemand: true,
    discardSelector: ".discard-answer"
    ,immediatelyShowMarkdownHelp:true
    });


    }
    });














    draft saved

    draft discarded


















    StackExchange.ready(
    function () {
    StackExchange.openid.initPostLogin('.new-post-login', 'https%3a%2f%2fmath.stackexchange.com%2fquestions%2f3065248%2fshow-that-lim-n-to-infty-int-01fxgnxdx-0-g-periodic-on-math%23new-answer', 'question_page');
    }
    );

    Post as a guest















    Required, but never shown

























    2 Answers
    2






    active

    oldest

    votes








    2 Answers
    2






    active

    oldest

    votes









    active

    oldest

    votes






    active

    oldest

    votes









    1












    $begingroup$

    For part (a):
    Let $I=int_{0}^{1}|g(x)|dx$. Let $epsilon>0$ be given. Note that
    $f$ is uniformly continuous on $[0,1]$, so there exists $delta>0$
    such that $|f(x)-f(y)|<frac{epsilon}{I+1}$ whenever $x,yin[0,1]$
    with $|x-y|<delta$. Choose $Ninmathbb{N}$ such that $frac{1}{N}<delta$.
    By considering the substitution $y=nx$, we may rewrite
    begin{eqnarray*}
    int_{0}^{1}f(x)g(nx)dx & = & frac{1}{n}int_{0}^{n}f(frac{y}{n})g(y)dy\
    & = & frac{1}{n}sum_{k=1}^{n}int_{k-1}^{k}f(frac{y}{n})g(y)dy.
    end{eqnarray*}

    Let $ngeq N$ be arbitrary. Denote $f_{k}=f(frac{k}{n})$, for $k=1,2,ldots n$.
    By periodicity of $g$, we have $int_{k-1}^{k}g(y)dy=int_{0}^{1}g(y)dy=0$.
    Therefore
    begin{eqnarray*}
    & & |int_{k-1}^{k}f(frac{y}{n})g(y)dy|\
    & = & |int_{k-1}^{k}f(frac{y}{n})-f_{k}g(y)dy|\
    & leq & int_{k-1}^{k}|f(frac{y}{n})-f_{k}||g(y)|dy\
    & leq & int_{k-1}^{k}frac{epsilon}{I+1}|g(y)|dy\
    & = & frac{epsilon}{I+1}int_{0}^{1}|g(y)|dy\
    & = & epsiloncdotfrac{I}{I+1}\
    & leq & epsilon.
    end{eqnarray*}

    Now it is clear that
    begin{eqnarray*}
    & & |int_{0}^{1}f(x)g(nx)dx|\
    & leq & frac{1}{n}sum_{k=1}^{n}|int_{k-1}^{k}f(frac{y}{n})g(y)dy|\
    & leq & frac{1}{n}sum_{k=1}^{n}epsilon\
    & = & epsilon.
    end{eqnarray*}






    share|cite|improve this answer









    $endgroup$













    • $begingroup$
      how you obtain this:begin{eqnarray*} & & |int_{k-1}^{k}f(frac{y}{n})g(y)dy|\ & = & |int_{k-1}^{k}f(frac{y}{n})-f_{k}g(y)dy| *****\ & leq & int_{k-1}^{k}|f(frac{y}{n})-f_{k}||g(y)|dy\ & leq & int_{k-1}^{k}frac{epsilon}{I+1}|g(y)|dy\ & = & frac{epsilon}{I+1}int_{0}^{1}|g(y)|dy\ & = & epsiloncdotfrac{I}{I+1}\ & leq & epsilon. end{eqnarray*}
      $endgroup$
      – Richard Clare
      Jan 7 at 18:05












    • $begingroup$
      $int_{k-1}^k f_k g(y) dy =0$
      $endgroup$
      – Danny Pak-Keung Chan
      Jan 7 at 18:11










    • $begingroup$
      Sure! Thanks!!!!!!!!
      $endgroup$
      – Richard Clare
      Jan 7 at 18:14
















    1












    $begingroup$

    For part (a):
    Let $I=int_{0}^{1}|g(x)|dx$. Let $epsilon>0$ be given. Note that
    $f$ is uniformly continuous on $[0,1]$, so there exists $delta>0$
    such that $|f(x)-f(y)|<frac{epsilon}{I+1}$ whenever $x,yin[0,1]$
    with $|x-y|<delta$. Choose $Ninmathbb{N}$ such that $frac{1}{N}<delta$.
    By considering the substitution $y=nx$, we may rewrite
    begin{eqnarray*}
    int_{0}^{1}f(x)g(nx)dx & = & frac{1}{n}int_{0}^{n}f(frac{y}{n})g(y)dy\
    & = & frac{1}{n}sum_{k=1}^{n}int_{k-1}^{k}f(frac{y}{n})g(y)dy.
    end{eqnarray*}

    Let $ngeq N$ be arbitrary. Denote $f_{k}=f(frac{k}{n})$, for $k=1,2,ldots n$.
    By periodicity of $g$, we have $int_{k-1}^{k}g(y)dy=int_{0}^{1}g(y)dy=0$.
    Therefore
    begin{eqnarray*}
    & & |int_{k-1}^{k}f(frac{y}{n})g(y)dy|\
    & = & |int_{k-1}^{k}f(frac{y}{n})-f_{k}g(y)dy|\
    & leq & int_{k-1}^{k}|f(frac{y}{n})-f_{k}||g(y)|dy\
    & leq & int_{k-1}^{k}frac{epsilon}{I+1}|g(y)|dy\
    & = & frac{epsilon}{I+1}int_{0}^{1}|g(y)|dy\
    & = & epsiloncdotfrac{I}{I+1}\
    & leq & epsilon.
    end{eqnarray*}

    Now it is clear that
    begin{eqnarray*}
    & & |int_{0}^{1}f(x)g(nx)dx|\
    & leq & frac{1}{n}sum_{k=1}^{n}|int_{k-1}^{k}f(frac{y}{n})g(y)dy|\
    & leq & frac{1}{n}sum_{k=1}^{n}epsilon\
    & = & epsilon.
    end{eqnarray*}






    share|cite|improve this answer









    $endgroup$













    • $begingroup$
      how you obtain this:begin{eqnarray*} & & |int_{k-1}^{k}f(frac{y}{n})g(y)dy|\ & = & |int_{k-1}^{k}f(frac{y}{n})-f_{k}g(y)dy| *****\ & leq & int_{k-1}^{k}|f(frac{y}{n})-f_{k}||g(y)|dy\ & leq & int_{k-1}^{k}frac{epsilon}{I+1}|g(y)|dy\ & = & frac{epsilon}{I+1}int_{0}^{1}|g(y)|dy\ & = & epsiloncdotfrac{I}{I+1}\ & leq & epsilon. end{eqnarray*}
      $endgroup$
      – Richard Clare
      Jan 7 at 18:05












    • $begingroup$
      $int_{k-1}^k f_k g(y) dy =0$
      $endgroup$
      – Danny Pak-Keung Chan
      Jan 7 at 18:11










    • $begingroup$
      Sure! Thanks!!!!!!!!
      $endgroup$
      – Richard Clare
      Jan 7 at 18:14














    1












    1








    1





    $begingroup$

    For part (a):
    Let $I=int_{0}^{1}|g(x)|dx$. Let $epsilon>0$ be given. Note that
    $f$ is uniformly continuous on $[0,1]$, so there exists $delta>0$
    such that $|f(x)-f(y)|<frac{epsilon}{I+1}$ whenever $x,yin[0,1]$
    with $|x-y|<delta$. Choose $Ninmathbb{N}$ such that $frac{1}{N}<delta$.
    By considering the substitution $y=nx$, we may rewrite
    begin{eqnarray*}
    int_{0}^{1}f(x)g(nx)dx & = & frac{1}{n}int_{0}^{n}f(frac{y}{n})g(y)dy\
    & = & frac{1}{n}sum_{k=1}^{n}int_{k-1}^{k}f(frac{y}{n})g(y)dy.
    end{eqnarray*}

    Let $ngeq N$ be arbitrary. Denote $f_{k}=f(frac{k}{n})$, for $k=1,2,ldots n$.
    By periodicity of $g$, we have $int_{k-1}^{k}g(y)dy=int_{0}^{1}g(y)dy=0$.
    Therefore
    begin{eqnarray*}
    & & |int_{k-1}^{k}f(frac{y}{n})g(y)dy|\
    & = & |int_{k-1}^{k}f(frac{y}{n})-f_{k}g(y)dy|\
    & leq & int_{k-1}^{k}|f(frac{y}{n})-f_{k}||g(y)|dy\
    & leq & int_{k-1}^{k}frac{epsilon}{I+1}|g(y)|dy\
    & = & frac{epsilon}{I+1}int_{0}^{1}|g(y)|dy\
    & = & epsiloncdotfrac{I}{I+1}\
    & leq & epsilon.
    end{eqnarray*}

    Now it is clear that
    begin{eqnarray*}
    & & |int_{0}^{1}f(x)g(nx)dx|\
    & leq & frac{1}{n}sum_{k=1}^{n}|int_{k-1}^{k}f(frac{y}{n})g(y)dy|\
    & leq & frac{1}{n}sum_{k=1}^{n}epsilon\
    & = & epsilon.
    end{eqnarray*}






    share|cite|improve this answer









    $endgroup$



    For part (a):
    Let $I=int_{0}^{1}|g(x)|dx$. Let $epsilon>0$ be given. Note that
    $f$ is uniformly continuous on $[0,1]$, so there exists $delta>0$
    such that $|f(x)-f(y)|<frac{epsilon}{I+1}$ whenever $x,yin[0,1]$
    with $|x-y|<delta$. Choose $Ninmathbb{N}$ such that $frac{1}{N}<delta$.
    By considering the substitution $y=nx$, we may rewrite
    begin{eqnarray*}
    int_{0}^{1}f(x)g(nx)dx & = & frac{1}{n}int_{0}^{n}f(frac{y}{n})g(y)dy\
    & = & frac{1}{n}sum_{k=1}^{n}int_{k-1}^{k}f(frac{y}{n})g(y)dy.
    end{eqnarray*}

    Let $ngeq N$ be arbitrary. Denote $f_{k}=f(frac{k}{n})$, for $k=1,2,ldots n$.
    By periodicity of $g$, we have $int_{k-1}^{k}g(y)dy=int_{0}^{1}g(y)dy=0$.
    Therefore
    begin{eqnarray*}
    & & |int_{k-1}^{k}f(frac{y}{n})g(y)dy|\
    & = & |int_{k-1}^{k}f(frac{y}{n})-f_{k}g(y)dy|\
    & leq & int_{k-1}^{k}|f(frac{y}{n})-f_{k}||g(y)|dy\
    & leq & int_{k-1}^{k}frac{epsilon}{I+1}|g(y)|dy\
    & = & frac{epsilon}{I+1}int_{0}^{1}|g(y)|dy\
    & = & epsiloncdotfrac{I}{I+1}\
    & leq & epsilon.
    end{eqnarray*}

    Now it is clear that
    begin{eqnarray*}
    & & |int_{0}^{1}f(x)g(nx)dx|\
    & leq & frac{1}{n}sum_{k=1}^{n}|int_{k-1}^{k}f(frac{y}{n})g(y)dy|\
    & leq & frac{1}{n}sum_{k=1}^{n}epsilon\
    & = & epsilon.
    end{eqnarray*}







    share|cite|improve this answer












    share|cite|improve this answer



    share|cite|improve this answer










    answered Jan 7 at 17:39









    Danny Pak-Keung ChanDanny Pak-Keung Chan

    2,32538




    2,32538












    • $begingroup$
      how you obtain this:begin{eqnarray*} & & |int_{k-1}^{k}f(frac{y}{n})g(y)dy|\ & = & |int_{k-1}^{k}f(frac{y}{n})-f_{k}g(y)dy| *****\ & leq & int_{k-1}^{k}|f(frac{y}{n})-f_{k}||g(y)|dy\ & leq & int_{k-1}^{k}frac{epsilon}{I+1}|g(y)|dy\ & = & frac{epsilon}{I+1}int_{0}^{1}|g(y)|dy\ & = & epsiloncdotfrac{I}{I+1}\ & leq & epsilon. end{eqnarray*}
      $endgroup$
      – Richard Clare
      Jan 7 at 18:05












    • $begingroup$
      $int_{k-1}^k f_k g(y) dy =0$
      $endgroup$
      – Danny Pak-Keung Chan
      Jan 7 at 18:11










    • $begingroup$
      Sure! Thanks!!!!!!!!
      $endgroup$
      – Richard Clare
      Jan 7 at 18:14


















    • $begingroup$
      how you obtain this:begin{eqnarray*} & & |int_{k-1}^{k}f(frac{y}{n})g(y)dy|\ & = & |int_{k-1}^{k}f(frac{y}{n})-f_{k}g(y)dy| *****\ & leq & int_{k-1}^{k}|f(frac{y}{n})-f_{k}||g(y)|dy\ & leq & int_{k-1}^{k}frac{epsilon}{I+1}|g(y)|dy\ & = & frac{epsilon}{I+1}int_{0}^{1}|g(y)|dy\ & = & epsiloncdotfrac{I}{I+1}\ & leq & epsilon. end{eqnarray*}
      $endgroup$
      – Richard Clare
      Jan 7 at 18:05












    • $begingroup$
      $int_{k-1}^k f_k g(y) dy =0$
      $endgroup$
      – Danny Pak-Keung Chan
      Jan 7 at 18:11










    • $begingroup$
      Sure! Thanks!!!!!!!!
      $endgroup$
      – Richard Clare
      Jan 7 at 18:14
















    $begingroup$
    how you obtain this:begin{eqnarray*} & & |int_{k-1}^{k}f(frac{y}{n})g(y)dy|\ & = & |int_{k-1}^{k}f(frac{y}{n})-f_{k}g(y)dy| *****\ & leq & int_{k-1}^{k}|f(frac{y}{n})-f_{k}||g(y)|dy\ & leq & int_{k-1}^{k}frac{epsilon}{I+1}|g(y)|dy\ & = & frac{epsilon}{I+1}int_{0}^{1}|g(y)|dy\ & = & epsiloncdotfrac{I}{I+1}\ & leq & epsilon. end{eqnarray*}
    $endgroup$
    – Richard Clare
    Jan 7 at 18:05






    $begingroup$
    how you obtain this:begin{eqnarray*} & & |int_{k-1}^{k}f(frac{y}{n})g(y)dy|\ & = & |int_{k-1}^{k}f(frac{y}{n})-f_{k}g(y)dy| *****\ & leq & int_{k-1}^{k}|f(frac{y}{n})-f_{k}||g(y)|dy\ & leq & int_{k-1}^{k}frac{epsilon}{I+1}|g(y)|dy\ & = & frac{epsilon}{I+1}int_{0}^{1}|g(y)|dy\ & = & epsiloncdotfrac{I}{I+1}\ & leq & epsilon. end{eqnarray*}
    $endgroup$
    – Richard Clare
    Jan 7 at 18:05














    $begingroup$
    $int_{k-1}^k f_k g(y) dy =0$
    $endgroup$
    – Danny Pak-Keung Chan
    Jan 7 at 18:11




    $begingroup$
    $int_{k-1}^k f_k g(y) dy =0$
    $endgroup$
    – Danny Pak-Keung Chan
    Jan 7 at 18:11












    $begingroup$
    Sure! Thanks!!!!!!!!
    $endgroup$
    – Richard Clare
    Jan 7 at 18:14




    $begingroup$
    Sure! Thanks!!!!!!!!
    $endgroup$
    – Richard Clare
    Jan 7 at 18:14











    1












    $begingroup$

    For (b): Since $g$ is continuous, there exists $M>0$ such that $|g(x)|leq M$
    for all $xin[0,1]$. By periodicity of $g$, it follows that $|g(x)|leq M$
    for all $xinmathbb{R}$. Recall that Lebesgue integrable function
    can be approximated by integrable continuous function in $||cdot||_{1}$-norm.
    More precisely, given $epsilon>0$, there exists a continuous function
    $h:mathbb{R}rightarrowmathbb{R}$ such that $int_{0}^{1}|f-h|<epsilon$.



    Let $epsilon>0$ be given. Choose continuous function $h:mathbb{R}rightarrowmathbb{R}$
    such that $int_{0}^{1}|f-h|<frac{epsilon}{2M}.$ By part (a), there
    exists $N$ such that $|int_{0}^{1}h(x)g(nx)dx|<frac{epsilon}{2}$
    whenever $ngeq N$. Now let $ngeq N$ be aribitrary, then
    begin{eqnarray*}
    & & |int_{0}^{1}f(x)g(nx)dx|\
    & leq & |int_{0}^{1}[f(x)-h(x)]g(nx)dx|+|int_{0}^{1}h(x)g(nx)dx|\
    & leq & Mint_{0}^{1}|f(x)-h(x)|dx+frac{epsilon}{2}\
    & < & epsilon.
    end{eqnarray*}






    share|cite|improve this answer









    $endgroup$


















      1












      $begingroup$

      For (b): Since $g$ is continuous, there exists $M>0$ such that $|g(x)|leq M$
      for all $xin[0,1]$. By periodicity of $g$, it follows that $|g(x)|leq M$
      for all $xinmathbb{R}$. Recall that Lebesgue integrable function
      can be approximated by integrable continuous function in $||cdot||_{1}$-norm.
      More precisely, given $epsilon>0$, there exists a continuous function
      $h:mathbb{R}rightarrowmathbb{R}$ such that $int_{0}^{1}|f-h|<epsilon$.



      Let $epsilon>0$ be given. Choose continuous function $h:mathbb{R}rightarrowmathbb{R}$
      such that $int_{0}^{1}|f-h|<frac{epsilon}{2M}.$ By part (a), there
      exists $N$ such that $|int_{0}^{1}h(x)g(nx)dx|<frac{epsilon}{2}$
      whenever $ngeq N$. Now let $ngeq N$ be aribitrary, then
      begin{eqnarray*}
      & & |int_{0}^{1}f(x)g(nx)dx|\
      & leq & |int_{0}^{1}[f(x)-h(x)]g(nx)dx|+|int_{0}^{1}h(x)g(nx)dx|\
      & leq & Mint_{0}^{1}|f(x)-h(x)|dx+frac{epsilon}{2}\
      & < & epsilon.
      end{eqnarray*}






      share|cite|improve this answer









      $endgroup$
















        1












        1








        1





        $begingroup$

        For (b): Since $g$ is continuous, there exists $M>0$ such that $|g(x)|leq M$
        for all $xin[0,1]$. By periodicity of $g$, it follows that $|g(x)|leq M$
        for all $xinmathbb{R}$. Recall that Lebesgue integrable function
        can be approximated by integrable continuous function in $||cdot||_{1}$-norm.
        More precisely, given $epsilon>0$, there exists a continuous function
        $h:mathbb{R}rightarrowmathbb{R}$ such that $int_{0}^{1}|f-h|<epsilon$.



        Let $epsilon>0$ be given. Choose continuous function $h:mathbb{R}rightarrowmathbb{R}$
        such that $int_{0}^{1}|f-h|<frac{epsilon}{2M}.$ By part (a), there
        exists $N$ such that $|int_{0}^{1}h(x)g(nx)dx|<frac{epsilon}{2}$
        whenever $ngeq N$. Now let $ngeq N$ be aribitrary, then
        begin{eqnarray*}
        & & |int_{0}^{1}f(x)g(nx)dx|\
        & leq & |int_{0}^{1}[f(x)-h(x)]g(nx)dx|+|int_{0}^{1}h(x)g(nx)dx|\
        & leq & Mint_{0}^{1}|f(x)-h(x)|dx+frac{epsilon}{2}\
        & < & epsilon.
        end{eqnarray*}






        share|cite|improve this answer









        $endgroup$



        For (b): Since $g$ is continuous, there exists $M>0$ such that $|g(x)|leq M$
        for all $xin[0,1]$. By periodicity of $g$, it follows that $|g(x)|leq M$
        for all $xinmathbb{R}$. Recall that Lebesgue integrable function
        can be approximated by integrable continuous function in $||cdot||_{1}$-norm.
        More precisely, given $epsilon>0$, there exists a continuous function
        $h:mathbb{R}rightarrowmathbb{R}$ such that $int_{0}^{1}|f-h|<epsilon$.



        Let $epsilon>0$ be given. Choose continuous function $h:mathbb{R}rightarrowmathbb{R}$
        such that $int_{0}^{1}|f-h|<frac{epsilon}{2M}.$ By part (a), there
        exists $N$ such that $|int_{0}^{1}h(x)g(nx)dx|<frac{epsilon}{2}$
        whenever $ngeq N$. Now let $ngeq N$ be aribitrary, then
        begin{eqnarray*}
        & & |int_{0}^{1}f(x)g(nx)dx|\
        & leq & |int_{0}^{1}[f(x)-h(x)]g(nx)dx|+|int_{0}^{1}h(x)g(nx)dx|\
        & leq & Mint_{0}^{1}|f(x)-h(x)|dx+frac{epsilon}{2}\
        & < & epsilon.
        end{eqnarray*}







        share|cite|improve this answer












        share|cite|improve this answer



        share|cite|improve this answer










        answered Jan 7 at 17:57









        Danny Pak-Keung ChanDanny Pak-Keung Chan

        2,32538




        2,32538






























            draft saved

            draft discarded




















































            Thanks for contributing an answer to Mathematics Stack Exchange!


            • Please be sure to answer the question. Provide details and share your research!

            But avoid



            • Asking for help, clarification, or responding to other answers.

            • Making statements based on opinion; back them up with references or personal experience.


            Use MathJax to format equations. MathJax reference.


            To learn more, see our tips on writing great answers.




            draft saved


            draft discarded














            StackExchange.ready(
            function () {
            StackExchange.openid.initPostLogin('.new-post-login', 'https%3a%2f%2fmath.stackexchange.com%2fquestions%2f3065248%2fshow-that-lim-n-to-infty-int-01fxgnxdx-0-g-periodic-on-math%23new-answer', 'question_page');
            }
            );

            Post as a guest















            Required, but never shown





















































            Required, but never shown














            Required, but never shown












            Required, but never shown







            Required, but never shown

































            Required, but never shown














            Required, but never shown












            Required, but never shown







            Required, but never shown







            Popular posts from this blog

            An IMO inspired problem

            Management

            Investment